Diễn Đàn MathScopeDiễn Đàn MathScope
  Diễn Đàn MathScope
Ghi Danh Hỏi/Ðáp Community Lịch

Go Back   Diễn Đàn MathScope > Đại Học Và Sau Đại Học/College Playground > Giải Tích/Analysis

News & Announcements

Ngoài một số quy định đã được nêu trong phần Quy định của Ghi Danh , mọi người tranh thủ bỏ ra 5 phút để đọc thêm một số Quy định sau để khỏi bị treo nick ở MathScope nhé !

* Nội quy MathScope.Org

* Một số quy định chung !

* Quy định về việc viết bài trong diễn đàn MathScope

* Nếu bạn muốn gia nhập đội ngũ BQT thì vui lòng tham gia tại đây

* Những câu hỏi thường gặp

* Về việc viết bài trong Box Đại học và Sau đại học


Trả lời Gởi Ðề Tài Mới
 
Ðiều Chỉnh Xếp Bài
Old 30-05-2012, 05:30 PM   #1
gd1468
+Thành Viên+
 
Tham gia ngày: May 2012
Bài gởi: 1
Thanks: 0
Thanked 0 Times in 0 Posts
$\sum_{n=1}^{\infty }\left ( \frac{3n-2}{3n+2} \right )^{n^{2}}$

Xét sự hội tụ của chuỗi số:

$$\sum_{n=1}^{\infty }\left ( \frac{3n-2}{3n+2} \right )^{n^{2}}$ $

Thầy mình hướng dẫn áp dụng "Căn số Cauchy" , mình làm theo nhưng giải ra 1. Mà trong định luật "Căn số Cauchy" thì 1 không xác định, chỉ có >1 hoặc <1 thôi. Các bạn giúp mình với, thanks !
[RIGHT][I][B]Nguồn: MathScope.ORG[/B][/I][/RIGHT]
 
gd1468 is offline   Trả Lời Với Trích Dẫn
Old 30-05-2012, 05:38 PM   #2
99
+Thành Viên+
 
Tham gia ngày: Nov 2007
Bài gởi: 2,995
Thanks: 537
Thanked 2,429 Times in 1,376 Posts
Hạng tử chuỗi của bạn có dạng $\left(1-\frac{1}{n}\right)^{n^2}$, có gì đó rất giống số $e.$ Cách tiếp cận là cố gắng so sánh hạng tử này với $\displaystyle\frac{1}{n^{\alpha}}$ (với $\alpha$ là hằng số thích hợp).

Bạn thử xét hàm $\displaystyle\left(1-x\right)^{\frac{1}{x^2}}$ với $0< x < \epsilon$ đủ nhỏ (khi khảo sát hàm này nhớ lấy logarithm, không là toi đấy ).
[RIGHT][I][B]Nguồn: MathScope.ORG[/B][/I][/RIGHT]
 
99 is offline   Trả Lời Với Trích Dẫn
Old 30-05-2012, 06:16 PM   #3
tr.phuoctoan
+Thành Viên+
 
tr.phuoctoan's Avatar
 
Tham gia ngày: Oct 2011
Bài gởi: 36
Thanks: 10
Thanked 7 Times in 5 Posts
Trích:
Nguyên văn bởi gd1468 View Post
Xét sự hội tụ của chuỗi số:

$$\sum_{n=1}^{\infty }\left ( \frac{3n-2}{3n+2} \right )^{n^{2}}$ $

Thầy mình hướng dẫn áp dụng "Căn số Cauchy" , mình làm theo nhưng giải ra 1. Mà trong định luật "Căn số Cauchy" thì 1 không xác định, chỉ có >1 hoặc <1 thôi. Các bạn giúp mình với, thanks !
Bạn kiểm tra lại xem giới hạn $ \left (\frac{3n-2}{3n+2}\right )^{n} $ khi $n \longrightarrow \infty $ có thật bằng 1 không ???
[RIGHT][I][B]Nguồn: MathScope.ORG[/B][/I][/RIGHT]
 
__________________
$--------------Tr.PhuocToan------ $
tr.phuoctoan is offline   Trả Lời Với Trích Dẫn
Old 30-05-2012, 06:47 PM   #4
coban
+Thành Viên+
 
coban's Avatar
 
Tham gia ngày: Nov 2009
Bài gởi: 174
Thanks: 110
Thanked 55 Times in 43 Posts
Trích:
Nguyên văn bởi gd1468 View Post
Xét sự hội tụ của chuỗi số:

$$\sum_{n=1}^{\infty }\left ( \frac{3n-2}{3n+2} \right )^{n^{2}}$ $

Thầy mình hướng dẫn áp dụng "Căn số Cauchy" , mình làm theo nhưng giải ra 1. Mà trong định luật "Căn số Cauchy" thì 1 không xác định, chỉ có >1 hoặc <1 thôi. Các bạn giúp mình với, thanks !
Bạn áp dụng dấu hiệu căn Cauchy sai rồi , ta có:
$\lim_{n\to\infty}\sqrt[n]{a_n} = \lim_{n\to\infty}\left ( \frac{3n-2}{3n+2} \right )^{n} = \lim_{n\to\infty}\left(1-\frac{4}{3n+2}\right)^n = \lim_{n\to\infty}\left[\left(1-\frac{4}{3n+2}\right)^{\frac{-3n-2}{4}}\right]^{\frac{-4n}{3n+2}}= e^{\frac{-4}{3}} < 1 $
Suy ra chuỗi đã cho hội tụ
[RIGHT][I][B]Nguồn: MathScope.ORG[/B][/I][/RIGHT]
 
__________________
Pure mathematics is, in its way, the poetry of logical ideas.
coban is offline   Trả Lời Với Trích Dẫn
Trả lời Gởi Ðề Tài Mới

Bookmarks


Quuyền Hạn Của Bạn
You may not post new threads
You may not post replies
You may not post attachments
You may not edit your posts

BB code is Mở
Smilies đang Mở
[IMG] đang Mở
HTML đang Tắt

Chuyển đến


Múi giờ GMT. Hiện tại là 06:55 PM.


Powered by: vBulletin Copyright ©2000-2024, Jelsoft Enterprises Ltd.
Inactive Reminders By mathscope.org
[page compression: 52.03 k/57.89 k (10.11%)]